Physics for Scientists and Engineers: A Strategic Approach with Modern Physics (3rd Edition)

Published by Pearson
ISBN 10: 0321740904
ISBN 13: 978-0-32174-090-8

Chapter 27 - Gauss's Law - Exercises and Problems - Page 809: 58

Answer

See the detailed answer below.

Work Step by Step

$$\color{blue}{\bf [a]}$$ We are given that the colume charge density is $$\rho =\rho_0\left[ 1-\dfrac{r}{R}\right] \tag 1$$ where $\rho_0$ is constant. Now we need to solve for $\rho_0$, $$dq=\rho dV$$ where $dV$ is the volume of a spherical shell of radius $r$, and thickness $dr$. Hence, $dV=(4\pi r^2) dr$. Hence, $$dq=\rho (4\pi r^2) dr$$ Plug $\rho $ from (1), $$dq=\rho_0\left[ 1-\dfrac{r}{R}\right] (4\pi r^2) dr \tag 2$$ Integrating; $$\int_0^Qdq=\int_0^R\rho_0\left[ 1-\dfrac{r}{R}\right] (4\pi r^2) dr $$ $$Q=4\pi\rho_0\int_0^R \left[ r^2-\dfrac{r^3}{R}\right] dr $$ $$Q=4\pi\rho_0 \left[ \dfrac{r^3}{3}-\dfrac{r^4}{4R}\right]_0^R $$ $$Q=4\pi\rho_0 \left[ \dfrac{R^3}{3}-\dfrac{R^4}{4R}\right] $$ $$Q=\dfrac{\pi\rho_0R^3}{3} $$ $$\boxed{\rho_0=\dfrac{3Q}{\pi R^3} }$$ $$\color{blue}{\bf [b]}$$ when $r\lt R$, $$\oint \vec E\cdot d\vec A=\dfrac{Q_{in}}{\epsilon_0}$$ $$ E A =\dfrac{Q_{in}}{\epsilon_0}$$ $$ E =\dfrac{Q_{in}}{A \epsilon_0}$$ $$ E =\dfrac{Q_{in}}{4\pi r^2\epsilon_0}\tag 3$$ Now we need to find $Q_{in}$, $$Q_{in}=\int dq$$ Plug from (2), $$Q_{in}=\int_0^{r} \rho_0\left[ 1-\dfrac{r}{R}\right] (4\pi r^2) dr $$ $$Q_{in}=4\pi \rho_0\int_0^{r} \left[ r^2-\dfrac{r^3}{R}\right] dr $$ $$Q_{in}=4\pi\rho_0 \left[ \dfrac{r^3}{3}-\dfrac{r^4}{4R}\right]_0^r $$ $$Q_{in}=4\pi\rho_0 \left[ \dfrac{r^3}{3}-\dfrac{r^4}{4R}\right] $$ $$Q_{in}=4\pi\rho_0 \left[ \dfrac{4Rr^3-3r^4}{12R}\right] $$ Plug $\rho_0$ from the boxed formula, $$Q_{in}=4\pi\dfrac{3Q}{\pi R^3} \left[ \dfrac{4Rr^3-3r^4}{12R}\right] $$ $$Q_{in}= \dfrac{ Q}{ R^3} \left[ \dfrac{4Rr^3-3r^4}{R}\right] $$ Plug into (3), $$ E =\dfrac{1}{4\pi r^2\epsilon_0} \dfrac{ Q}{ R^3} \left[ \dfrac{4Rr^3-3r^4}{R}\right] $$ $$ E =\dfrac{Q}{(4\pi \epsilon_0)R^3} \left[ {4 r -\dfrac{3r^2}{R}} \right] $$ $$ E =\dfrac{Qr}{(4\pi \epsilon_0)R^3} \left[ {4 -\dfrac{3r }{R}} \right] $$ Therefore, $$ \boxed{E =\dfrac{Qr}{(4\pi \epsilon_0)R^3} \left[ {4 -\dfrac{3r }{R}} \right]\;\hat r} $$ the direction is outward since the magnitude of the electric field is positive since $r\lt R$. $$\color{blue}{\bf [c]}$$ when $r=R$, $$ E =\dfrac{QR}{(4\pi \epsilon_0)R^3} \left[ {4 -\dfrac{3R }{R}} \right] =\dfrac{Q }{(4\pi \epsilon_0)R^2} $$ which is a reasonable result since this is the electric field of a point charge at distance $R$ since the spherical charge behaves as if the entire charge were at the center when $r\geq R$.
Update this answer!

You can help us out by revising, improving and updating this answer.

Update this answer

After you claim an answer you’ll have 24 hours to send in a draft. An editor will review the submission and either publish your submission or provide feedback.